LSAT and Law School Admissions Forum

Get expert LSAT preparation and law school admissions advice from PowerScore Test Preparation.

 Administrator
PowerScore Staff
  • PowerScore Staff
  • Posts: 8916
  • Joined: Feb 02, 2011
|
#36570
Complete Question Explanation
(See the complete passage discussion here: lsat/viewtopic.php?t=14542)

The correct answer choice is (B)


This question asks for an overview of the structure of the passage. As prephrased in the discussion
of the structure above, this passage introduces a prevailing view, and then refutes it with empirical
evidence and suggests developing a different perspective.

Answer choice (A): Although the author does indeed take issue with an assertion, the author does not
speculate as to why the proponents are supportive, so this answer choice is incorrect.

Answer choice (B): This is the correct answer choice, as it restates the prephrased answer
discussed above. The author takes issue with a common assertion among communication specialists,
offers examples which show that the assertion in some cases runs counter to empirical fact, and
suggests a more anthropologist-like approach, with two specific issues to consider.

Answer choice (C): This answer choice correctly asserts that the author takes issue with an assertion.
The author, however, then introduces examples that prove that the referenced assertion sometimes
run counter to factual evidence, rather than presenting a new view and supporting it with examples.
The author then offers two considerations relevant to the development of a new view.

Answer choice (D): This answer choice is correct inasmuch as the author takes issue with an
assertion, presents counter examples, and suggests a change to the conventional approach. However,
the author then discusses two considerations relevant to the new approach, rather than two benefits to
the proponents, so this answer choice is incorrect.

Answer choice (E): The author does take issue with an assertion, and present examples that run
counter to that assertion. And although the author does suggest considerations for the development
of a new view, the author never suggests a compromise between the old assertion and the new view,
so this answer choice is incorrect.

Get the most out of your LSAT Prep Plus subscription.

Analyze and track your performance with our Testing and Analytics Package.